What are examples of books which teach the practice of mathematics?

The name of the pictureThe name of the pictureThe name of the pictureClash Royale CLAN TAG#URR8PPP











up vote
48
down vote

favorite
43












One may classify the types of mathematics books written for students into two groups: books which merely teach mathematics (i.e., they present theorems and proofs, ready-made, as it were) and those books which teach one the art of mathematics (i.e., they do not merely teach one mathematical results, but also how to do mathematics). Obviously books of the latter type are harder to write and, consequently, are rare. However, I've been reading W. W. Sawyer recently and have thoroughly enjoyed his writing (even those parts that covered material I'd known before -- known albeit in the sense of 'type one' books).



Naturally, one seeks to find out other books of the sort. Therefore, if you know books that fall in the latter category, please recommend them. To be clear, this book should not merely present theorems and perhaps require the reader to supply some dry, routine proof like induction, or some such thing; but it should lead the reader, as it were, into doing mathematics; that is, the book should explain why results are needed (in other words, provide sufficient motivation), how they might have been originally discovered (before being polished into an artificial presentation, say) and explain other contextual information of relevance to a particular mathematical result. To be further clear, I have read Polya and Euler, for example, and they both seem to fall in this category too. There should definitely be others that I'm missing. Please do recommend them. Many thanks.










share|cite|improve this question























  • How about this book?
    – user170039
    Sep 2 at 13:28






  • 2




    Thurston’s “On Proof and Progress in Mathematics” is not a book, but still a classic text on this topic. arxiv.org/abs/math/9404236
    – Matt F.
    Sep 3 at 5:25






  • 1




    There is a related question at math.stackexchange.com/questions/828458/….
    – mweiss
    Sep 3 at 15:31














up vote
48
down vote

favorite
43












One may classify the types of mathematics books written for students into two groups: books which merely teach mathematics (i.e., they present theorems and proofs, ready-made, as it were) and those books which teach one the art of mathematics (i.e., they do not merely teach one mathematical results, but also how to do mathematics). Obviously books of the latter type are harder to write and, consequently, are rare. However, I've been reading W. W. Sawyer recently and have thoroughly enjoyed his writing (even those parts that covered material I'd known before -- known albeit in the sense of 'type one' books).



Naturally, one seeks to find out other books of the sort. Therefore, if you know books that fall in the latter category, please recommend them. To be clear, this book should not merely present theorems and perhaps require the reader to supply some dry, routine proof like induction, or some such thing; but it should lead the reader, as it were, into doing mathematics; that is, the book should explain why results are needed (in other words, provide sufficient motivation), how they might have been originally discovered (before being polished into an artificial presentation, say) and explain other contextual information of relevance to a particular mathematical result. To be further clear, I have read Polya and Euler, for example, and they both seem to fall in this category too. There should definitely be others that I'm missing. Please do recommend them. Many thanks.










share|cite|improve this question























  • How about this book?
    – user170039
    Sep 2 at 13:28






  • 2




    Thurston’s “On Proof and Progress in Mathematics” is not a book, but still a classic text on this topic. arxiv.org/abs/math/9404236
    – Matt F.
    Sep 3 at 5:25






  • 1




    There is a related question at math.stackexchange.com/questions/828458/….
    – mweiss
    Sep 3 at 15:31












up vote
48
down vote

favorite
43









up vote
48
down vote

favorite
43






43





One may classify the types of mathematics books written for students into two groups: books which merely teach mathematics (i.e., they present theorems and proofs, ready-made, as it were) and those books which teach one the art of mathematics (i.e., they do not merely teach one mathematical results, but also how to do mathematics). Obviously books of the latter type are harder to write and, consequently, are rare. However, I've been reading W. W. Sawyer recently and have thoroughly enjoyed his writing (even those parts that covered material I'd known before -- known albeit in the sense of 'type one' books).



Naturally, one seeks to find out other books of the sort. Therefore, if you know books that fall in the latter category, please recommend them. To be clear, this book should not merely present theorems and perhaps require the reader to supply some dry, routine proof like induction, or some such thing; but it should lead the reader, as it were, into doing mathematics; that is, the book should explain why results are needed (in other words, provide sufficient motivation), how they might have been originally discovered (before being polished into an artificial presentation, say) and explain other contextual information of relevance to a particular mathematical result. To be further clear, I have read Polya and Euler, for example, and they both seem to fall in this category too. There should definitely be others that I'm missing. Please do recommend them. Many thanks.










share|cite|improve this question















One may classify the types of mathematics books written for students into two groups: books which merely teach mathematics (i.e., they present theorems and proofs, ready-made, as it were) and those books which teach one the art of mathematics (i.e., they do not merely teach one mathematical results, but also how to do mathematics). Obviously books of the latter type are harder to write and, consequently, are rare. However, I've been reading W. W. Sawyer recently and have thoroughly enjoyed his writing (even those parts that covered material I'd known before -- known albeit in the sense of 'type one' books).



Naturally, one seeks to find out other books of the sort. Therefore, if you know books that fall in the latter category, please recommend them. To be clear, this book should not merely present theorems and perhaps require the reader to supply some dry, routine proof like induction, or some such thing; but it should lead the reader, as it were, into doing mathematics; that is, the book should explain why results are needed (in other words, provide sufficient motivation), how they might have been originally discovered (before being polished into an artificial presentation, say) and explain other contextual information of relevance to a particular mathematical result. To be further clear, I have read Polya and Euler, for example, and they both seem to fall in this category too. There should definitely be others that I'm missing. Please do recommend them. Many thanks.







reference-request soft-question books advice heuristics






share|cite|improve this question















share|cite|improve this question













share|cite|improve this question




share|cite|improve this question








edited Sep 2 at 10:57


























community wiki





Allawonder












  • How about this book?
    – user170039
    Sep 2 at 13:28






  • 2




    Thurston’s “On Proof and Progress in Mathematics” is not a book, but still a classic text on this topic. arxiv.org/abs/math/9404236
    – Matt F.
    Sep 3 at 5:25






  • 1




    There is a related question at math.stackexchange.com/questions/828458/….
    – mweiss
    Sep 3 at 15:31
















  • How about this book?
    – user170039
    Sep 2 at 13:28






  • 2




    Thurston’s “On Proof and Progress in Mathematics” is not a book, but still a classic text on this topic. arxiv.org/abs/math/9404236
    – Matt F.
    Sep 3 at 5:25






  • 1




    There is a related question at math.stackexchange.com/questions/828458/….
    – mweiss
    Sep 3 at 15:31















How about this book?
– user170039
Sep 2 at 13:28




How about this book?
– user170039
Sep 2 at 13:28




2




2




Thurston’s “On Proof and Progress in Mathematics” is not a book, but still a classic text on this topic. arxiv.org/abs/math/9404236
– Matt F.
Sep 3 at 5:25




Thurston’s “On Proof and Progress in Mathematics” is not a book, but still a classic text on this topic. arxiv.org/abs/math/9404236
– Matt F.
Sep 3 at 5:25




1




1




There is a related question at math.stackexchange.com/questions/828458/….
– mweiss
Sep 3 at 15:31




There is a related question at math.stackexchange.com/questions/828458/….
– mweiss
Sep 3 at 15:31










14 Answers
14






active

oldest

votes

















up vote
21
down vote













On my opinion, the prime examples are the books of Polya:



  1. Polya and Szego, Problems and theorems in analysis (level: graduate+).


  2. Polya, Mathematics and plausible reasoning (all levels)


  3. Polya, Mathematical discovery (high school to undergraduate)


  4. Polya, How to solve it (high school).






share|cite|improve this answer


















  • 2




    "How to solve it" was incredibly unhelpful to me, as well as most of the high school friends I have met at various camps. It lays out a method of problem-solving and proof, but really only explicitly states what any rational person would try to do when solving a problem. I think it's the sort of book that a professional mathematician would read and love because it puts into words what you do when proving, but is frustrating to anyone trying to practically use and apply it because the most crucial part of proof is the intuition gained by practice, for which there are no "3 easy steps".
    – TreFox
    Sep 2 at 16:49






  • 1




    That being said, once you have built enough of that intuition, it's a good reality check.
    – TreFox
    Sep 2 at 16:50










  • @TreFox: I have to confess that "How to solve it" is the only book of the list which I have not read:-) But I can certify the usefulness of 1,2(first part) and 3.
    – Alexandre Eremenko
    Sep 3 at 3:46






  • 1




    the only one i have read parts of is #4. and from it i remember only one piece of advice, but one which would have fattened my vita had i heeded it earlier in my research career, namely (roughy from memory) "solutions to problems are like grapes, they come in bunches. so when you solve one problem look aound for another one that will yield to the same idea."
    – roy smith
    Sep 6 at 22:22

















up vote
8
down vote













Two books by David Bressoud:



  1. Bressoud, David M. Proofs and Confirmations. The story of the alternating sign matrix conjecture. MAA Spectrum. 1999


"My intention in this book is not just to describe this discovery of new mathematics, but to guide you into this land and to lead you up some of the recently scaled peaks. This is not an exhaustive account of all the marvels that have been uncovered, but rather a selected tour that will, I hope, encourage you to return and pursue your own explorations.''




There are conjectures; partial progress on the conjectures leads to new conjectures... students could learn something about the practice of mathematics by reading this book.



  1. Bressoud, David M. A radical approach to real analysis. Classroom Resource Materials Series, 2. MAA. 1994.

From Math Reviews:




This book "starts with infinite series, illustrating the great successes that led the early pioneers onward, as well as the obstacles that stymied even such luminaries as Euler and Lagrange''. Mistakes that were made are emphasized to highlight difficult concepts, and the student is led through some of the evolution of such concepts as uniform convergence as they arose in response to a need.







share|cite|improve this answer





























    up vote
    7
    down vote













    A few of my favourites that immediately come to mind:



    • Janich, Klaus. Topology

    • Carter, Nathan. Visual group theory

    • Lawvere, F. William and Stephen H. Schanuel. Conceptual mathematics: a first introduction to categories

    • Aigner, Martin; Ziegler, Günter. Proofs from THE BOOK

    Also, in their own weird way:



    • Steen, Lynn Arthur and Seebach, J. Arthur. Counterexamples in topology

    • Nelsen, Roger B. Proofs without words: Exercises in visual thinking (series)





    share|cite|improve this answer






















    • I wouldn't say Janich's Topology teaches how to do math. It's a great book, that conveys the "wonder" of mathematics, but I think it hardly teaches anything about how to "do" it.
      – Qfwfq
      Sep 2 at 10:06






    • 2




      I have to disagree: he gives plenty of visual intuition about a subject that is far too often presented in an almost purely algebraic way. I have found that visual intuition enormously helpful whenever I had to plot a high-level course through a topology proof. Imho there should be many more textbooks like that.
      – Stefano Gogioso
      Sep 2 at 12:04







    • 3




      @StefanoGogioso, topology is often presented purely algebraically?
      – LSpice
      Sep 2 at 14:40






    • 5




      @LSpice I suspect "algebra" here is in the sense of Boolean (etc.) algebra: that is, spaces are discussed as if they were just families of sets with certain properties, rather than geometric objects.
      – Noah Schweber
      Sep 2 at 23:33

















    up vote
    7
    down vote













    • How to prove it, by Daniel Velleman.


    • Einführung in das mathematische Arbeiten, by Hermann Schichl and Roland Steinbauer. (In German)


    Both are on a very elementary level.






    share|cite|improve this answer





























      up vote
      7
      down vote













      When I was in college, I read “The Psychology of Invention in the Mathematical Field” by Hadamard, and I found it impressive. My first reaction today is to say that the book nonetheless did not teach me to do mathematics. On further consideration, I think that maybe it taught me the value of down time: leaving the problem in the back of my mind while I did entirely different things; and the value of sleep.






      share|cite|improve this answer





























        up vote
        4
        down vote













        It not clear to me what kind of book your looking for, or at what level. Or perhaps it would be better to say I don't believe that you have a dichotomous classification into books that only teach mathematical content versus how to do mathematics.



        At a very elementary level, one book which hasn't been mentioned yet is




        • How to think like a mathematician, by Kevin Houston

        that I've sometimes used as a supplementary book in an intro to proofs course.



        Then after one progresses to a more advanced level, I think one learns the art of mathematics not so much by explicit meta-construction, but by seeing it and figuring it out oneself. That said, there are some books which help with this more than others, and here are a few more specialized ones that I think are good:



        • Course in arithmetic (or anything) by JP Serre (he's very concise and on the surface you might place this in your first category, the presentation and choice of material is excellent, and I think the process of reading Serre and figuring out the details helps one's mathematical maturity greatly)


        • Problems in Algebraic Number Theory by Murty and Esmonde, or similar books in this vein (there are some basic definitions, and then a load of exercises (with hints and solutions at the end) for you to develop the theory on your own, a quasi-Moore method sort of thing)


        • Foundations of Algebraic Geometry notes by Ravi Vakil (he has lots of meta-mathematical notes on why you do things a certain way, that I think help mature one's mathematical philosophy)






        share|cite|improve this answer


















        • 1




          To reply to your first paragraph; an example of a well-written work of art that does not pretend to teach the art of mathematics is that classic by Gauss, Disquisitiones Arithmeticae. Thanks for your recommendations, too.
          – Allawonder
          Sep 2 at 15:33

















        up vote
        4
        down vote













        Although it sits at the interface between maths and physics, Gauge Fields, Knots & Gravity by Baez and Munian can be in the list. It finally made Differential geometry click for me as it provided much of the physical and mathematical motivation behind the concepts.






        share|cite|improve this answer





























          up vote
          3
          down vote













          How about:



          Daniel Solow, How to Read and Do Proofs






          share|cite|improve this answer





























            up vote
            2
            down vote













            This is not so much "how to do mathematics" (to quote the OP),
            but more how it feels to do mathematics.
            A New Yorker review said this book "is less about math than about mathematicians—how they live, how they work, and how they talk to one another." I found it inspiring.




            Villani, Cédric. Birth of a Theorem: A Mathematical Adventure. Farrar, Straus and Giroux, 2015.
            (Guardian review.
            AMS review.)




                     
            Villani



            The AMS review ends: "The implicit task that Villani had set for himself, of explaining what it is all about, is a difficult one .... To my mind the book succeeds wonderfully."






            share|cite|improve this answer


















            • 2




              A somewhat countervailing opinion was offered in a comment by Joel Adler here: mathoverflow.net/questions/297632/… I haven't read it myself, but I guess YMMV.
              – Todd Trimble♦
              Sep 2 at 17:07











            • I liked the Villani book, but it has little relevance to the question posed here... any more than A beautiful mind teaches one how to create deep theorems in Game Theory or differential equations.
              – David G. Stork
              Sep 6 at 20:24

















            up vote
            1
            down vote













            I may be not the only one, but I learned all my practice of working mathematician within the treatise of N. Bourbaki (Theory of sets, Algebra, General topology, Functions of a real variable, Topological vector spaces, Integration, Lie groups and Lie algebras, Commutative algebra, Spectral theories). The front address "To the reader" helped me to use theory, exercises, counter-examples, bibliography. It is a very clear guide (remains to work).






            share|cite|improve this answer





























              up vote
              1
              down vote













              This may not be what you are looking for but I really enjoyed the Probability Lifesaver by Steven J. Miller. It is very thorough, emphasizes the importance of proofs and leads the reader through those proofs, fairly rigorous while leaving out some of the finer details, and helps to think through non-intuitive problems in probability theory. The style is also very accessible and readable.



              The emphasis on proofs was especially appreciated. I am often tempted to skip proofs since I use probability theory for application and am not a mathematician. A lot of the proofs in this book are described in a style that tries to justify why one would try a particular strategy at each step. The author leads the reader through the proof and tries different approaches, even ones that lead to failure, for the purpose of instruction. This was extremely useful.



              I enjoyed this book because it gave me the confidence and appreciation of math that has made me more comfortable in reading more advanced and rigorous texts.






              share|cite|improve this answer





























                up vote
                0
                down vote













                Probably at more elementary level than you intend, but otherwise (I think) remarkably on target,




                • Burn, R. P., Numbers and functions: steps into analysis., Cambridge: Cambridge Univ. Press. xix, 328 p. (1993). ZBL0872.00009.

                (I wish I remembered where I recently saw it recommended. This related question mentions it along with others by the same author.)






                share|cite|improve this answer





























                  up vote
                  0
                  down vote













                  Numbers and Functions by Victor H. Moll and The Concrete Tetrahedron by Manuel Kauers and Peter Paule on an elementary level and Advanced Determinant Calculus (Séminaire Lotharingien Combin. 42 (1999) by Christian Krattenthaler on an advanced level.






                  share|cite|improve this answer





























                    up vote
                    -1
                    down vote













                    If German is an option as well, a rather new book on the market, supplied with more than 1000 youtube videos:



                    Konkrete Mathematik nicht nur für Informatiker






                    share|cite|improve this answer






















                      Your Answer




                      StackExchange.ifUsing("editor", function ()
                      return StackExchange.using("mathjaxEditing", function ()
                      StackExchange.MarkdownEditor.creationCallbacks.add(function (editor, postfix)
                      StackExchange.mathjaxEditing.prepareWmdForMathJax(editor, postfix, [["$", "$"], ["\\(","\\)"]]);
                      );
                      );
                      , "mathjax-editing");

                      StackExchange.ready(function()
                      var channelOptions =
                      tags: "".split(" "),
                      id: "504"
                      ;
                      initTagRenderer("".split(" "), "".split(" "), channelOptions);

                      StackExchange.using("externalEditor", function()
                      // Have to fire editor after snippets, if snippets enabled
                      if (StackExchange.settings.snippets.snippetsEnabled)
                      StackExchange.using("snippets", function()
                      createEditor();
                      );

                      else
                      createEditor();

                      );

                      function createEditor()
                      StackExchange.prepareEditor(
                      heartbeatType: 'answer',
                      convertImagesToLinks: true,
                      noModals: false,
                      showLowRepImageUploadWarning: true,
                      reputationToPostImages: 10,
                      bindNavPrevention: true,
                      postfix: "",
                      noCode: true, onDemand: true,
                      discardSelector: ".discard-answer"
                      ,immediatelyShowMarkdownHelp:true
                      );



                      );













                       

                      draft saved


                      draft discarded


















                      StackExchange.ready(
                      function ()
                      StackExchange.openid.initPostLogin('.new-post-login', 'https%3a%2f%2fmathoverflow.net%2fquestions%2f309650%2fwhat-are-examples-of-books-which-teach-the-practice-of-mathematics%23new-answer', 'question_page');

                      );

                      Post as a guest






























                      14 Answers
                      14






                      active

                      oldest

                      votes








                      14 Answers
                      14






                      active

                      oldest

                      votes









                      active

                      oldest

                      votes






                      active

                      oldest

                      votes








                      up vote
                      21
                      down vote













                      On my opinion, the prime examples are the books of Polya:



                      1. Polya and Szego, Problems and theorems in analysis (level: graduate+).


                      2. Polya, Mathematics and plausible reasoning (all levels)


                      3. Polya, Mathematical discovery (high school to undergraduate)


                      4. Polya, How to solve it (high school).






                      share|cite|improve this answer


















                      • 2




                        "How to solve it" was incredibly unhelpful to me, as well as most of the high school friends I have met at various camps. It lays out a method of problem-solving and proof, but really only explicitly states what any rational person would try to do when solving a problem. I think it's the sort of book that a professional mathematician would read and love because it puts into words what you do when proving, but is frustrating to anyone trying to practically use and apply it because the most crucial part of proof is the intuition gained by practice, for which there are no "3 easy steps".
                        – TreFox
                        Sep 2 at 16:49






                      • 1




                        That being said, once you have built enough of that intuition, it's a good reality check.
                        – TreFox
                        Sep 2 at 16:50










                      • @TreFox: I have to confess that "How to solve it" is the only book of the list which I have not read:-) But I can certify the usefulness of 1,2(first part) and 3.
                        – Alexandre Eremenko
                        Sep 3 at 3:46






                      • 1




                        the only one i have read parts of is #4. and from it i remember only one piece of advice, but one which would have fattened my vita had i heeded it earlier in my research career, namely (roughy from memory) "solutions to problems are like grapes, they come in bunches. so when you solve one problem look aound for another one that will yield to the same idea."
                        – roy smith
                        Sep 6 at 22:22














                      up vote
                      21
                      down vote













                      On my opinion, the prime examples are the books of Polya:



                      1. Polya and Szego, Problems and theorems in analysis (level: graduate+).


                      2. Polya, Mathematics and plausible reasoning (all levels)


                      3. Polya, Mathematical discovery (high school to undergraduate)


                      4. Polya, How to solve it (high school).






                      share|cite|improve this answer


















                      • 2




                        "How to solve it" was incredibly unhelpful to me, as well as most of the high school friends I have met at various camps. It lays out a method of problem-solving and proof, but really only explicitly states what any rational person would try to do when solving a problem. I think it's the sort of book that a professional mathematician would read and love because it puts into words what you do when proving, but is frustrating to anyone trying to practically use and apply it because the most crucial part of proof is the intuition gained by practice, for which there are no "3 easy steps".
                        – TreFox
                        Sep 2 at 16:49






                      • 1




                        That being said, once you have built enough of that intuition, it's a good reality check.
                        – TreFox
                        Sep 2 at 16:50










                      • @TreFox: I have to confess that "How to solve it" is the only book of the list which I have not read:-) But I can certify the usefulness of 1,2(first part) and 3.
                        – Alexandre Eremenko
                        Sep 3 at 3:46






                      • 1




                        the only one i have read parts of is #4. and from it i remember only one piece of advice, but one which would have fattened my vita had i heeded it earlier in my research career, namely (roughy from memory) "solutions to problems are like grapes, they come in bunches. so when you solve one problem look aound for another one that will yield to the same idea."
                        – roy smith
                        Sep 6 at 22:22












                      up vote
                      21
                      down vote










                      up vote
                      21
                      down vote









                      On my opinion, the prime examples are the books of Polya:



                      1. Polya and Szego, Problems and theorems in analysis (level: graduate+).


                      2. Polya, Mathematics and plausible reasoning (all levels)


                      3. Polya, Mathematical discovery (high school to undergraduate)


                      4. Polya, How to solve it (high school).






                      share|cite|improve this answer














                      On my opinion, the prime examples are the books of Polya:



                      1. Polya and Szego, Problems and theorems in analysis (level: graduate+).


                      2. Polya, Mathematics and plausible reasoning (all levels)


                      3. Polya, Mathematical discovery (high school to undergraduate)


                      4. Polya, How to solve it (high school).







                      share|cite|improve this answer














                      share|cite|improve this answer



                      share|cite|improve this answer








                      answered Sep 2 at 13:35


























                      community wiki





                      Alexandre Eremenko








                      • 2




                        "How to solve it" was incredibly unhelpful to me, as well as most of the high school friends I have met at various camps. It lays out a method of problem-solving and proof, but really only explicitly states what any rational person would try to do when solving a problem. I think it's the sort of book that a professional mathematician would read and love because it puts into words what you do when proving, but is frustrating to anyone trying to practically use and apply it because the most crucial part of proof is the intuition gained by practice, for which there are no "3 easy steps".
                        – TreFox
                        Sep 2 at 16:49






                      • 1




                        That being said, once you have built enough of that intuition, it's a good reality check.
                        – TreFox
                        Sep 2 at 16:50










                      • @TreFox: I have to confess that "How to solve it" is the only book of the list which I have not read:-) But I can certify the usefulness of 1,2(first part) and 3.
                        – Alexandre Eremenko
                        Sep 3 at 3:46






                      • 1




                        the only one i have read parts of is #4. and from it i remember only one piece of advice, but one which would have fattened my vita had i heeded it earlier in my research career, namely (roughy from memory) "solutions to problems are like grapes, they come in bunches. so when you solve one problem look aound for another one that will yield to the same idea."
                        – roy smith
                        Sep 6 at 22:22












                      • 2




                        "How to solve it" was incredibly unhelpful to me, as well as most of the high school friends I have met at various camps. It lays out a method of problem-solving and proof, but really only explicitly states what any rational person would try to do when solving a problem. I think it's the sort of book that a professional mathematician would read and love because it puts into words what you do when proving, but is frustrating to anyone trying to practically use and apply it because the most crucial part of proof is the intuition gained by practice, for which there are no "3 easy steps".
                        – TreFox
                        Sep 2 at 16:49






                      • 1




                        That being said, once you have built enough of that intuition, it's a good reality check.
                        – TreFox
                        Sep 2 at 16:50










                      • @TreFox: I have to confess that "How to solve it" is the only book of the list which I have not read:-) But I can certify the usefulness of 1,2(first part) and 3.
                        – Alexandre Eremenko
                        Sep 3 at 3:46






                      • 1




                        the only one i have read parts of is #4. and from it i remember only one piece of advice, but one which would have fattened my vita had i heeded it earlier in my research career, namely (roughy from memory) "solutions to problems are like grapes, they come in bunches. so when you solve one problem look aound for another one that will yield to the same idea."
                        – roy smith
                        Sep 6 at 22:22







                      2




                      2




                      "How to solve it" was incredibly unhelpful to me, as well as most of the high school friends I have met at various camps. It lays out a method of problem-solving and proof, but really only explicitly states what any rational person would try to do when solving a problem. I think it's the sort of book that a professional mathematician would read and love because it puts into words what you do when proving, but is frustrating to anyone trying to practically use and apply it because the most crucial part of proof is the intuition gained by practice, for which there are no "3 easy steps".
                      – TreFox
                      Sep 2 at 16:49




                      "How to solve it" was incredibly unhelpful to me, as well as most of the high school friends I have met at various camps. It lays out a method of problem-solving and proof, but really only explicitly states what any rational person would try to do when solving a problem. I think it's the sort of book that a professional mathematician would read and love because it puts into words what you do when proving, but is frustrating to anyone trying to practically use and apply it because the most crucial part of proof is the intuition gained by practice, for which there are no "3 easy steps".
                      – TreFox
                      Sep 2 at 16:49




                      1




                      1




                      That being said, once you have built enough of that intuition, it's a good reality check.
                      – TreFox
                      Sep 2 at 16:50




                      That being said, once you have built enough of that intuition, it's a good reality check.
                      – TreFox
                      Sep 2 at 16:50












                      @TreFox: I have to confess that "How to solve it" is the only book of the list which I have not read:-) But I can certify the usefulness of 1,2(first part) and 3.
                      – Alexandre Eremenko
                      Sep 3 at 3:46




                      @TreFox: I have to confess that "How to solve it" is the only book of the list which I have not read:-) But I can certify the usefulness of 1,2(first part) and 3.
                      – Alexandre Eremenko
                      Sep 3 at 3:46




                      1




                      1




                      the only one i have read parts of is #4. and from it i remember only one piece of advice, but one which would have fattened my vita had i heeded it earlier in my research career, namely (roughy from memory) "solutions to problems are like grapes, they come in bunches. so when you solve one problem look aound for another one that will yield to the same idea."
                      – roy smith
                      Sep 6 at 22:22




                      the only one i have read parts of is #4. and from it i remember only one piece of advice, but one which would have fattened my vita had i heeded it earlier in my research career, namely (roughy from memory) "solutions to problems are like grapes, they come in bunches. so when you solve one problem look aound for another one that will yield to the same idea."
                      – roy smith
                      Sep 6 at 22:22










                      up vote
                      8
                      down vote













                      Two books by David Bressoud:



                      1. Bressoud, David M. Proofs and Confirmations. The story of the alternating sign matrix conjecture. MAA Spectrum. 1999


                      "My intention in this book is not just to describe this discovery of new mathematics, but to guide you into this land and to lead you up some of the recently scaled peaks. This is not an exhaustive account of all the marvels that have been uncovered, but rather a selected tour that will, I hope, encourage you to return and pursue your own explorations.''




                      There are conjectures; partial progress on the conjectures leads to new conjectures... students could learn something about the practice of mathematics by reading this book.



                      1. Bressoud, David M. A radical approach to real analysis. Classroom Resource Materials Series, 2. MAA. 1994.

                      From Math Reviews:




                      This book "starts with infinite series, illustrating the great successes that led the early pioneers onward, as well as the obstacles that stymied even such luminaries as Euler and Lagrange''. Mistakes that were made are emphasized to highlight difficult concepts, and the student is led through some of the evolution of such concepts as uniform convergence as they arose in response to a need.







                      share|cite|improve this answer


























                        up vote
                        8
                        down vote













                        Two books by David Bressoud:



                        1. Bressoud, David M. Proofs and Confirmations. The story of the alternating sign matrix conjecture. MAA Spectrum. 1999


                        "My intention in this book is not just to describe this discovery of new mathematics, but to guide you into this land and to lead you up some of the recently scaled peaks. This is not an exhaustive account of all the marvels that have been uncovered, but rather a selected tour that will, I hope, encourage you to return and pursue your own explorations.''




                        There are conjectures; partial progress on the conjectures leads to new conjectures... students could learn something about the practice of mathematics by reading this book.



                        1. Bressoud, David M. A radical approach to real analysis. Classroom Resource Materials Series, 2. MAA. 1994.

                        From Math Reviews:




                        This book "starts with infinite series, illustrating the great successes that led the early pioneers onward, as well as the obstacles that stymied even such luminaries as Euler and Lagrange''. Mistakes that were made are emphasized to highlight difficult concepts, and the student is led through some of the evolution of such concepts as uniform convergence as they arose in response to a need.







                        share|cite|improve this answer
























                          up vote
                          8
                          down vote










                          up vote
                          8
                          down vote









                          Two books by David Bressoud:



                          1. Bressoud, David M. Proofs and Confirmations. The story of the alternating sign matrix conjecture. MAA Spectrum. 1999


                          "My intention in this book is not just to describe this discovery of new mathematics, but to guide you into this land and to lead you up some of the recently scaled peaks. This is not an exhaustive account of all the marvels that have been uncovered, but rather a selected tour that will, I hope, encourage you to return and pursue your own explorations.''




                          There are conjectures; partial progress on the conjectures leads to new conjectures... students could learn something about the practice of mathematics by reading this book.



                          1. Bressoud, David M. A radical approach to real analysis. Classroom Resource Materials Series, 2. MAA. 1994.

                          From Math Reviews:




                          This book "starts with infinite series, illustrating the great successes that led the early pioneers onward, as well as the obstacles that stymied even such luminaries as Euler and Lagrange''. Mistakes that were made are emphasized to highlight difficult concepts, and the student is led through some of the evolution of such concepts as uniform convergence as they arose in response to a need.







                          share|cite|improve this answer














                          Two books by David Bressoud:



                          1. Bressoud, David M. Proofs and Confirmations. The story of the alternating sign matrix conjecture. MAA Spectrum. 1999


                          "My intention in this book is not just to describe this discovery of new mathematics, but to guide you into this land and to lead you up some of the recently scaled peaks. This is not an exhaustive account of all the marvels that have been uncovered, but rather a selected tour that will, I hope, encourage you to return and pursue your own explorations.''




                          There are conjectures; partial progress on the conjectures leads to new conjectures... students could learn something about the practice of mathematics by reading this book.



                          1. Bressoud, David M. A radical approach to real analysis. Classroom Resource Materials Series, 2. MAA. 1994.

                          From Math Reviews:




                          This book "starts with infinite series, illustrating the great successes that led the early pioneers onward, as well as the obstacles that stymied even such luminaries as Euler and Lagrange''. Mistakes that were made are emphasized to highlight difficult concepts, and the student is led through some of the evolution of such concepts as uniform convergence as they arose in response to a need.








                          share|cite|improve this answer














                          share|cite|improve this answer



                          share|cite|improve this answer








                          answered Sep 2 at 17:30


























                          community wiki





                          Zach Teitler





















                              up vote
                              7
                              down vote













                              A few of my favourites that immediately come to mind:



                              • Janich, Klaus. Topology

                              • Carter, Nathan. Visual group theory

                              • Lawvere, F. William and Stephen H. Schanuel. Conceptual mathematics: a first introduction to categories

                              • Aigner, Martin; Ziegler, Günter. Proofs from THE BOOK

                              Also, in their own weird way:



                              • Steen, Lynn Arthur and Seebach, J. Arthur. Counterexamples in topology

                              • Nelsen, Roger B. Proofs without words: Exercises in visual thinking (series)





                              share|cite|improve this answer






















                              • I wouldn't say Janich's Topology teaches how to do math. It's a great book, that conveys the "wonder" of mathematics, but I think it hardly teaches anything about how to "do" it.
                                – Qfwfq
                                Sep 2 at 10:06






                              • 2




                                I have to disagree: he gives plenty of visual intuition about a subject that is far too often presented in an almost purely algebraic way. I have found that visual intuition enormously helpful whenever I had to plot a high-level course through a topology proof. Imho there should be many more textbooks like that.
                                – Stefano Gogioso
                                Sep 2 at 12:04







                              • 3




                                @StefanoGogioso, topology is often presented purely algebraically?
                                – LSpice
                                Sep 2 at 14:40






                              • 5




                                @LSpice I suspect "algebra" here is in the sense of Boolean (etc.) algebra: that is, spaces are discussed as if they were just families of sets with certain properties, rather than geometric objects.
                                – Noah Schweber
                                Sep 2 at 23:33














                              up vote
                              7
                              down vote













                              A few of my favourites that immediately come to mind:



                              • Janich, Klaus. Topology

                              • Carter, Nathan. Visual group theory

                              • Lawvere, F. William and Stephen H. Schanuel. Conceptual mathematics: a first introduction to categories

                              • Aigner, Martin; Ziegler, Günter. Proofs from THE BOOK

                              Also, in their own weird way:



                              • Steen, Lynn Arthur and Seebach, J. Arthur. Counterexamples in topology

                              • Nelsen, Roger B. Proofs without words: Exercises in visual thinking (series)





                              share|cite|improve this answer






















                              • I wouldn't say Janich's Topology teaches how to do math. It's a great book, that conveys the "wonder" of mathematics, but I think it hardly teaches anything about how to "do" it.
                                – Qfwfq
                                Sep 2 at 10:06






                              • 2




                                I have to disagree: he gives plenty of visual intuition about a subject that is far too often presented in an almost purely algebraic way. I have found that visual intuition enormously helpful whenever I had to plot a high-level course through a topology proof. Imho there should be many more textbooks like that.
                                – Stefano Gogioso
                                Sep 2 at 12:04







                              • 3




                                @StefanoGogioso, topology is often presented purely algebraically?
                                – LSpice
                                Sep 2 at 14:40






                              • 5




                                @LSpice I suspect "algebra" here is in the sense of Boolean (etc.) algebra: that is, spaces are discussed as if they were just families of sets with certain properties, rather than geometric objects.
                                – Noah Schweber
                                Sep 2 at 23:33












                              up vote
                              7
                              down vote










                              up vote
                              7
                              down vote









                              A few of my favourites that immediately come to mind:



                              • Janich, Klaus. Topology

                              • Carter, Nathan. Visual group theory

                              • Lawvere, F. William and Stephen H. Schanuel. Conceptual mathematics: a first introduction to categories

                              • Aigner, Martin; Ziegler, Günter. Proofs from THE BOOK

                              Also, in their own weird way:



                              • Steen, Lynn Arthur and Seebach, J. Arthur. Counterexamples in topology

                              • Nelsen, Roger B. Proofs without words: Exercises in visual thinking (series)





                              share|cite|improve this answer














                              A few of my favourites that immediately come to mind:



                              • Janich, Klaus. Topology

                              • Carter, Nathan. Visual group theory

                              • Lawvere, F. William and Stephen H. Schanuel. Conceptual mathematics: a first introduction to categories

                              • Aigner, Martin; Ziegler, Günter. Proofs from THE BOOK

                              Also, in their own weird way:



                              • Steen, Lynn Arthur and Seebach, J. Arthur. Counterexamples in topology

                              • Nelsen, Roger B. Proofs without words: Exercises in visual thinking (series)






                              share|cite|improve this answer














                              share|cite|improve this answer



                              share|cite|improve this answer








                              edited Sep 2 at 9:24


























                              community wiki





                              Stefano Gogioso












                              • I wouldn't say Janich's Topology teaches how to do math. It's a great book, that conveys the "wonder" of mathematics, but I think it hardly teaches anything about how to "do" it.
                                – Qfwfq
                                Sep 2 at 10:06






                              • 2




                                I have to disagree: he gives plenty of visual intuition about a subject that is far too often presented in an almost purely algebraic way. I have found that visual intuition enormously helpful whenever I had to plot a high-level course through a topology proof. Imho there should be many more textbooks like that.
                                – Stefano Gogioso
                                Sep 2 at 12:04







                              • 3




                                @StefanoGogioso, topology is often presented purely algebraically?
                                – LSpice
                                Sep 2 at 14:40






                              • 5




                                @LSpice I suspect "algebra" here is in the sense of Boolean (etc.) algebra: that is, spaces are discussed as if they were just families of sets with certain properties, rather than geometric objects.
                                – Noah Schweber
                                Sep 2 at 23:33
















                              • I wouldn't say Janich's Topology teaches how to do math. It's a great book, that conveys the "wonder" of mathematics, but I think it hardly teaches anything about how to "do" it.
                                – Qfwfq
                                Sep 2 at 10:06






                              • 2




                                I have to disagree: he gives plenty of visual intuition about a subject that is far too often presented in an almost purely algebraic way. I have found that visual intuition enormously helpful whenever I had to plot a high-level course through a topology proof. Imho there should be many more textbooks like that.
                                – Stefano Gogioso
                                Sep 2 at 12:04







                              • 3




                                @StefanoGogioso, topology is often presented purely algebraically?
                                – LSpice
                                Sep 2 at 14:40






                              • 5




                                @LSpice I suspect "algebra" here is in the sense of Boolean (etc.) algebra: that is, spaces are discussed as if they were just families of sets with certain properties, rather than geometric objects.
                                – Noah Schweber
                                Sep 2 at 23:33















                              I wouldn't say Janich's Topology teaches how to do math. It's a great book, that conveys the "wonder" of mathematics, but I think it hardly teaches anything about how to "do" it.
                              – Qfwfq
                              Sep 2 at 10:06




                              I wouldn't say Janich's Topology teaches how to do math. It's a great book, that conveys the "wonder" of mathematics, but I think it hardly teaches anything about how to "do" it.
                              – Qfwfq
                              Sep 2 at 10:06




                              2




                              2




                              I have to disagree: he gives plenty of visual intuition about a subject that is far too often presented in an almost purely algebraic way. I have found that visual intuition enormously helpful whenever I had to plot a high-level course through a topology proof. Imho there should be many more textbooks like that.
                              – Stefano Gogioso
                              Sep 2 at 12:04





                              I have to disagree: he gives plenty of visual intuition about a subject that is far too often presented in an almost purely algebraic way. I have found that visual intuition enormously helpful whenever I had to plot a high-level course through a topology proof. Imho there should be many more textbooks like that.
                              – Stefano Gogioso
                              Sep 2 at 12:04





                              3




                              3




                              @StefanoGogioso, topology is often presented purely algebraically?
                              – LSpice
                              Sep 2 at 14:40




                              @StefanoGogioso, topology is often presented purely algebraically?
                              – LSpice
                              Sep 2 at 14:40




                              5




                              5




                              @LSpice I suspect "algebra" here is in the sense of Boolean (etc.) algebra: that is, spaces are discussed as if they were just families of sets with certain properties, rather than geometric objects.
                              – Noah Schweber
                              Sep 2 at 23:33




                              @LSpice I suspect "algebra" here is in the sense of Boolean (etc.) algebra: that is, spaces are discussed as if they were just families of sets with certain properties, rather than geometric objects.
                              – Noah Schweber
                              Sep 2 at 23:33










                              up vote
                              7
                              down vote













                              • How to prove it, by Daniel Velleman.


                              • Einführung in das mathematische Arbeiten, by Hermann Schichl and Roland Steinbauer. (In German)


                              Both are on a very elementary level.






                              share|cite|improve this answer


























                                up vote
                                7
                                down vote













                                • How to prove it, by Daniel Velleman.


                                • Einführung in das mathematische Arbeiten, by Hermann Schichl and Roland Steinbauer. (In German)


                                Both are on a very elementary level.






                                share|cite|improve this answer
























                                  up vote
                                  7
                                  down vote










                                  up vote
                                  7
                                  down vote









                                  • How to prove it, by Daniel Velleman.


                                  • Einführung in das mathematische Arbeiten, by Hermann Schichl and Roland Steinbauer. (In German)


                                  Both are on a very elementary level.






                                  share|cite|improve this answer














                                  • How to prove it, by Daniel Velleman.


                                  • Einführung in das mathematische Arbeiten, by Hermann Schichl and Roland Steinbauer. (In German)


                                  Both are on a very elementary level.







                                  share|cite|improve this answer














                                  share|cite|improve this answer



                                  share|cite|improve this answer








                                  answered Sep 2 at 12:10


























                                  community wiki





                                  Goldstern





















                                      up vote
                                      7
                                      down vote













                                      When I was in college, I read “The Psychology of Invention in the Mathematical Field” by Hadamard, and I found it impressive. My first reaction today is to say that the book nonetheless did not teach me to do mathematics. On further consideration, I think that maybe it taught me the value of down time: leaving the problem in the back of my mind while I did entirely different things; and the value of sleep.






                                      share|cite|improve this answer


























                                        up vote
                                        7
                                        down vote













                                        When I was in college, I read “The Psychology of Invention in the Mathematical Field” by Hadamard, and I found it impressive. My first reaction today is to say that the book nonetheless did not teach me to do mathematics. On further consideration, I think that maybe it taught me the value of down time: leaving the problem in the back of my mind while I did entirely different things; and the value of sleep.






                                        share|cite|improve this answer
























                                          up vote
                                          7
                                          down vote










                                          up vote
                                          7
                                          down vote









                                          When I was in college, I read “The Psychology of Invention in the Mathematical Field” by Hadamard, and I found it impressive. My first reaction today is to say that the book nonetheless did not teach me to do mathematics. On further consideration, I think that maybe it taught me the value of down time: leaving the problem in the back of my mind while I did entirely different things; and the value of sleep.






                                          share|cite|improve this answer














                                          When I was in college, I read “The Psychology of Invention in the Mathematical Field” by Hadamard, and I found it impressive. My first reaction today is to say that the book nonetheless did not teach me to do mathematics. On further consideration, I think that maybe it taught me the value of down time: leaving the problem in the back of my mind while I did entirely different things; and the value of sleep.







                                          share|cite|improve this answer














                                          share|cite|improve this answer



                                          share|cite|improve this answer








                                          answered Sep 2 at 17:42


























                                          community wiki





                                          Lubin





















                                              up vote
                                              4
                                              down vote













                                              It not clear to me what kind of book your looking for, or at what level. Or perhaps it would be better to say I don't believe that you have a dichotomous classification into books that only teach mathematical content versus how to do mathematics.



                                              At a very elementary level, one book which hasn't been mentioned yet is




                                              • How to think like a mathematician, by Kevin Houston

                                              that I've sometimes used as a supplementary book in an intro to proofs course.



                                              Then after one progresses to a more advanced level, I think one learns the art of mathematics not so much by explicit meta-construction, but by seeing it and figuring it out oneself. That said, there are some books which help with this more than others, and here are a few more specialized ones that I think are good:



                                              • Course in arithmetic (or anything) by JP Serre (he's very concise and on the surface you might place this in your first category, the presentation and choice of material is excellent, and I think the process of reading Serre and figuring out the details helps one's mathematical maturity greatly)


                                              • Problems in Algebraic Number Theory by Murty and Esmonde, or similar books in this vein (there are some basic definitions, and then a load of exercises (with hints and solutions at the end) for you to develop the theory on your own, a quasi-Moore method sort of thing)


                                              • Foundations of Algebraic Geometry notes by Ravi Vakil (he has lots of meta-mathematical notes on why you do things a certain way, that I think help mature one's mathematical philosophy)






                                              share|cite|improve this answer


















                                              • 1




                                                To reply to your first paragraph; an example of a well-written work of art that does not pretend to teach the art of mathematics is that classic by Gauss, Disquisitiones Arithmeticae. Thanks for your recommendations, too.
                                                – Allawonder
                                                Sep 2 at 15:33














                                              up vote
                                              4
                                              down vote













                                              It not clear to me what kind of book your looking for, or at what level. Or perhaps it would be better to say I don't believe that you have a dichotomous classification into books that only teach mathematical content versus how to do mathematics.



                                              At a very elementary level, one book which hasn't been mentioned yet is




                                              • How to think like a mathematician, by Kevin Houston

                                              that I've sometimes used as a supplementary book in an intro to proofs course.



                                              Then after one progresses to a more advanced level, I think one learns the art of mathematics not so much by explicit meta-construction, but by seeing it and figuring it out oneself. That said, there are some books which help with this more than others, and here are a few more specialized ones that I think are good:



                                              • Course in arithmetic (or anything) by JP Serre (he's very concise and on the surface you might place this in your first category, the presentation and choice of material is excellent, and I think the process of reading Serre and figuring out the details helps one's mathematical maturity greatly)


                                              • Problems in Algebraic Number Theory by Murty and Esmonde, or similar books in this vein (there are some basic definitions, and then a load of exercises (with hints and solutions at the end) for you to develop the theory on your own, a quasi-Moore method sort of thing)


                                              • Foundations of Algebraic Geometry notes by Ravi Vakil (he has lots of meta-mathematical notes on why you do things a certain way, that I think help mature one's mathematical philosophy)






                                              share|cite|improve this answer


















                                              • 1




                                                To reply to your first paragraph; an example of a well-written work of art that does not pretend to teach the art of mathematics is that classic by Gauss, Disquisitiones Arithmeticae. Thanks for your recommendations, too.
                                                – Allawonder
                                                Sep 2 at 15:33












                                              up vote
                                              4
                                              down vote










                                              up vote
                                              4
                                              down vote









                                              It not clear to me what kind of book your looking for, or at what level. Or perhaps it would be better to say I don't believe that you have a dichotomous classification into books that only teach mathematical content versus how to do mathematics.



                                              At a very elementary level, one book which hasn't been mentioned yet is




                                              • How to think like a mathematician, by Kevin Houston

                                              that I've sometimes used as a supplementary book in an intro to proofs course.



                                              Then after one progresses to a more advanced level, I think one learns the art of mathematics not so much by explicit meta-construction, but by seeing it and figuring it out oneself. That said, there are some books which help with this more than others, and here are a few more specialized ones that I think are good:



                                              • Course in arithmetic (or anything) by JP Serre (he's very concise and on the surface you might place this in your first category, the presentation and choice of material is excellent, and I think the process of reading Serre and figuring out the details helps one's mathematical maturity greatly)


                                              • Problems in Algebraic Number Theory by Murty and Esmonde, or similar books in this vein (there are some basic definitions, and then a load of exercises (with hints and solutions at the end) for you to develop the theory on your own, a quasi-Moore method sort of thing)


                                              • Foundations of Algebraic Geometry notes by Ravi Vakil (he has lots of meta-mathematical notes on why you do things a certain way, that I think help mature one's mathematical philosophy)






                                              share|cite|improve this answer














                                              It not clear to me what kind of book your looking for, or at what level. Or perhaps it would be better to say I don't believe that you have a dichotomous classification into books that only teach mathematical content versus how to do mathematics.



                                              At a very elementary level, one book which hasn't been mentioned yet is




                                              • How to think like a mathematician, by Kevin Houston

                                              that I've sometimes used as a supplementary book in an intro to proofs course.



                                              Then after one progresses to a more advanced level, I think one learns the art of mathematics not so much by explicit meta-construction, but by seeing it and figuring it out oneself. That said, there are some books which help with this more than others, and here are a few more specialized ones that I think are good:



                                              • Course in arithmetic (or anything) by JP Serre (he's very concise and on the surface you might place this in your first category, the presentation and choice of material is excellent, and I think the process of reading Serre and figuring out the details helps one's mathematical maturity greatly)


                                              • Problems in Algebraic Number Theory by Murty and Esmonde, or similar books in this vein (there are some basic definitions, and then a load of exercises (with hints and solutions at the end) for you to develop the theory on your own, a quasi-Moore method sort of thing)


                                              • Foundations of Algebraic Geometry notes by Ravi Vakil (he has lots of meta-mathematical notes on why you do things a certain way, that I think help mature one's mathematical philosophy)







                                              share|cite|improve this answer














                                              share|cite|improve this answer



                                              share|cite|improve this answer








                                              answered Sep 2 at 14:47


























                                              community wiki





                                              Kimball








                                              • 1




                                                To reply to your first paragraph; an example of a well-written work of art that does not pretend to teach the art of mathematics is that classic by Gauss, Disquisitiones Arithmeticae. Thanks for your recommendations, too.
                                                – Allawonder
                                                Sep 2 at 15:33












                                              • 1




                                                To reply to your first paragraph; an example of a well-written work of art that does not pretend to teach the art of mathematics is that classic by Gauss, Disquisitiones Arithmeticae. Thanks for your recommendations, too.
                                                – Allawonder
                                                Sep 2 at 15:33







                                              1




                                              1




                                              To reply to your first paragraph; an example of a well-written work of art that does not pretend to teach the art of mathematics is that classic by Gauss, Disquisitiones Arithmeticae. Thanks for your recommendations, too.
                                              – Allawonder
                                              Sep 2 at 15:33




                                              To reply to your first paragraph; an example of a well-written work of art that does not pretend to teach the art of mathematics is that classic by Gauss, Disquisitiones Arithmeticae. Thanks for your recommendations, too.
                                              – Allawonder
                                              Sep 2 at 15:33










                                              up vote
                                              4
                                              down vote













                                              Although it sits at the interface between maths and physics, Gauge Fields, Knots & Gravity by Baez and Munian can be in the list. It finally made Differential geometry click for me as it provided much of the physical and mathematical motivation behind the concepts.






                                              share|cite|improve this answer


























                                                up vote
                                                4
                                                down vote













                                                Although it sits at the interface between maths and physics, Gauge Fields, Knots & Gravity by Baez and Munian can be in the list. It finally made Differential geometry click for me as it provided much of the physical and mathematical motivation behind the concepts.






                                                share|cite|improve this answer
























                                                  up vote
                                                  4
                                                  down vote










                                                  up vote
                                                  4
                                                  down vote









                                                  Although it sits at the interface between maths and physics, Gauge Fields, Knots & Gravity by Baez and Munian can be in the list. It finally made Differential geometry click for me as it provided much of the physical and mathematical motivation behind the concepts.






                                                  share|cite|improve this answer














                                                  Although it sits at the interface between maths and physics, Gauge Fields, Knots & Gravity by Baez and Munian can be in the list. It finally made Differential geometry click for me as it provided much of the physical and mathematical motivation behind the concepts.







                                                  share|cite|improve this answer














                                                  share|cite|improve this answer



                                                  share|cite|improve this answer








                                                  answered Sep 2 at 18:25


























                                                  community wiki





                                                  Andrea





















                                                      up vote
                                                      3
                                                      down vote













                                                      How about:



                                                      Daniel Solow, How to Read and Do Proofs






                                                      share|cite|improve this answer


























                                                        up vote
                                                        3
                                                        down vote













                                                        How about:



                                                        Daniel Solow, How to Read and Do Proofs






                                                        share|cite|improve this answer
























                                                          up vote
                                                          3
                                                          down vote










                                                          up vote
                                                          3
                                                          down vote









                                                          How about:



                                                          Daniel Solow, How to Read and Do Proofs






                                                          share|cite|improve this answer














                                                          How about:



                                                          Daniel Solow, How to Read and Do Proofs







                                                          share|cite|improve this answer














                                                          share|cite|improve this answer



                                                          share|cite|improve this answer








                                                          answered Sep 2 at 11:49


























                                                          community wiki





                                                          Gerald Edgar





















                                                              up vote
                                                              2
                                                              down vote













                                                              This is not so much "how to do mathematics" (to quote the OP),
                                                              but more how it feels to do mathematics.
                                                              A New Yorker review said this book "is less about math than about mathematicians—how they live, how they work, and how they talk to one another." I found it inspiring.




                                                              Villani, Cédric. Birth of a Theorem: A Mathematical Adventure. Farrar, Straus and Giroux, 2015.
                                                              (Guardian review.
                                                              AMS review.)




                                                                       
                                                              Villani



                                                              The AMS review ends: "The implicit task that Villani had set for himself, of explaining what it is all about, is a difficult one .... To my mind the book succeeds wonderfully."






                                                              share|cite|improve this answer


















                                                              • 2




                                                                A somewhat countervailing opinion was offered in a comment by Joel Adler here: mathoverflow.net/questions/297632/… I haven't read it myself, but I guess YMMV.
                                                                – Todd Trimble♦
                                                                Sep 2 at 17:07











                                                              • I liked the Villani book, but it has little relevance to the question posed here... any more than A beautiful mind teaches one how to create deep theorems in Game Theory or differential equations.
                                                                – David G. Stork
                                                                Sep 6 at 20:24














                                                              up vote
                                                              2
                                                              down vote













                                                              This is not so much "how to do mathematics" (to quote the OP),
                                                              but more how it feels to do mathematics.
                                                              A New Yorker review said this book "is less about math than about mathematicians—how they live, how they work, and how they talk to one another." I found it inspiring.




                                                              Villani, Cédric. Birth of a Theorem: A Mathematical Adventure. Farrar, Straus and Giroux, 2015.
                                                              (Guardian review.
                                                              AMS review.)




                                                                       
                                                              Villani



                                                              The AMS review ends: "The implicit task that Villani had set for himself, of explaining what it is all about, is a difficult one .... To my mind the book succeeds wonderfully."






                                                              share|cite|improve this answer


















                                                              • 2




                                                                A somewhat countervailing opinion was offered in a comment by Joel Adler here: mathoverflow.net/questions/297632/… I haven't read it myself, but I guess YMMV.
                                                                – Todd Trimble♦
                                                                Sep 2 at 17:07











                                                              • I liked the Villani book, but it has little relevance to the question posed here... any more than A beautiful mind teaches one how to create deep theorems in Game Theory or differential equations.
                                                                – David G. Stork
                                                                Sep 6 at 20:24












                                                              up vote
                                                              2
                                                              down vote










                                                              up vote
                                                              2
                                                              down vote









                                                              This is not so much "how to do mathematics" (to quote the OP),
                                                              but more how it feels to do mathematics.
                                                              A New Yorker review said this book "is less about math than about mathematicians—how they live, how they work, and how they talk to one another." I found it inspiring.




                                                              Villani, Cédric. Birth of a Theorem: A Mathematical Adventure. Farrar, Straus and Giroux, 2015.
                                                              (Guardian review.
                                                              AMS review.)




                                                                       
                                                              Villani



                                                              The AMS review ends: "The implicit task that Villani had set for himself, of explaining what it is all about, is a difficult one .... To my mind the book succeeds wonderfully."






                                                              share|cite|improve this answer














                                                              This is not so much "how to do mathematics" (to quote the OP),
                                                              but more how it feels to do mathematics.
                                                              A New Yorker review said this book "is less about math than about mathematicians—how they live, how they work, and how they talk to one another." I found it inspiring.




                                                              Villani, Cédric. Birth of a Theorem: A Mathematical Adventure. Farrar, Straus and Giroux, 2015.
                                                              (Guardian review.
                                                              AMS review.)




                                                                       
                                                              Villani



                                                              The AMS review ends: "The implicit task that Villani had set for himself, of explaining what it is all about, is a difficult one .... To my mind the book succeeds wonderfully."







                                                              share|cite|improve this answer














                                                              share|cite|improve this answer



                                                              share|cite|improve this answer








                                                              edited Sep 2 at 17:20


























                                                              community wiki





                                                              2 revs
                                                              Joseph O'Rourke








                                                              • 2




                                                                A somewhat countervailing opinion was offered in a comment by Joel Adler here: mathoverflow.net/questions/297632/… I haven't read it myself, but I guess YMMV.
                                                                – Todd Trimble♦
                                                                Sep 2 at 17:07











                                                              • I liked the Villani book, but it has little relevance to the question posed here... any more than A beautiful mind teaches one how to create deep theorems in Game Theory or differential equations.
                                                                – David G. Stork
                                                                Sep 6 at 20:24












                                                              • 2




                                                                A somewhat countervailing opinion was offered in a comment by Joel Adler here: mathoverflow.net/questions/297632/… I haven't read it myself, but I guess YMMV.
                                                                – Todd Trimble♦
                                                                Sep 2 at 17:07











                                                              • I liked the Villani book, but it has little relevance to the question posed here... any more than A beautiful mind teaches one how to create deep theorems in Game Theory or differential equations.
                                                                – David G. Stork
                                                                Sep 6 at 20:24







                                                              2




                                                              2




                                                              A somewhat countervailing opinion was offered in a comment by Joel Adler here: mathoverflow.net/questions/297632/… I haven't read it myself, but I guess YMMV.
                                                              – Todd Trimble♦
                                                              Sep 2 at 17:07





                                                              A somewhat countervailing opinion was offered in a comment by Joel Adler here: mathoverflow.net/questions/297632/… I haven't read it myself, but I guess YMMV.
                                                              – Todd Trimble♦
                                                              Sep 2 at 17:07













                                                              I liked the Villani book, but it has little relevance to the question posed here... any more than A beautiful mind teaches one how to create deep theorems in Game Theory or differential equations.
                                                              – David G. Stork
                                                              Sep 6 at 20:24




                                                              I liked the Villani book, but it has little relevance to the question posed here... any more than A beautiful mind teaches one how to create deep theorems in Game Theory or differential equations.
                                                              – David G. Stork
                                                              Sep 6 at 20:24










                                                              up vote
                                                              1
                                                              down vote













                                                              I may be not the only one, but I learned all my practice of working mathematician within the treatise of N. Bourbaki (Theory of sets, Algebra, General topology, Functions of a real variable, Topological vector spaces, Integration, Lie groups and Lie algebras, Commutative algebra, Spectral theories). The front address "To the reader" helped me to use theory, exercises, counter-examples, bibliography. It is a very clear guide (remains to work).






                                                              share|cite|improve this answer


























                                                                up vote
                                                                1
                                                                down vote













                                                                I may be not the only one, but I learned all my practice of working mathematician within the treatise of N. Bourbaki (Theory of sets, Algebra, General topology, Functions of a real variable, Topological vector spaces, Integration, Lie groups and Lie algebras, Commutative algebra, Spectral theories). The front address "To the reader" helped me to use theory, exercises, counter-examples, bibliography. It is a very clear guide (remains to work).






                                                                share|cite|improve this answer
























                                                                  up vote
                                                                  1
                                                                  down vote










                                                                  up vote
                                                                  1
                                                                  down vote









                                                                  I may be not the only one, but I learned all my practice of working mathematician within the treatise of N. Bourbaki (Theory of sets, Algebra, General topology, Functions of a real variable, Topological vector spaces, Integration, Lie groups and Lie algebras, Commutative algebra, Spectral theories). The front address "To the reader" helped me to use theory, exercises, counter-examples, bibliography. It is a very clear guide (remains to work).






                                                                  share|cite|improve this answer














                                                                  I may be not the only one, but I learned all my practice of working mathematician within the treatise of N. Bourbaki (Theory of sets, Algebra, General topology, Functions of a real variable, Topological vector spaces, Integration, Lie groups and Lie algebras, Commutative algebra, Spectral theories). The front address "To the reader" helped me to use theory, exercises, counter-examples, bibliography. It is a very clear guide (remains to work).







                                                                  share|cite|improve this answer














                                                                  share|cite|improve this answer



                                                                  share|cite|improve this answer








                                                                  answered Sep 2 at 18:20


























                                                                  community wiki





                                                                  Duchamp Gérard H. E.





















                                                                      up vote
                                                                      1
                                                                      down vote













                                                                      This may not be what you are looking for but I really enjoyed the Probability Lifesaver by Steven J. Miller. It is very thorough, emphasizes the importance of proofs and leads the reader through those proofs, fairly rigorous while leaving out some of the finer details, and helps to think through non-intuitive problems in probability theory. The style is also very accessible and readable.



                                                                      The emphasis on proofs was especially appreciated. I am often tempted to skip proofs since I use probability theory for application and am not a mathematician. A lot of the proofs in this book are described in a style that tries to justify why one would try a particular strategy at each step. The author leads the reader through the proof and tries different approaches, even ones that lead to failure, for the purpose of instruction. This was extremely useful.



                                                                      I enjoyed this book because it gave me the confidence and appreciation of math that has made me more comfortable in reading more advanced and rigorous texts.






                                                                      share|cite|improve this answer


























                                                                        up vote
                                                                        1
                                                                        down vote













                                                                        This may not be what you are looking for but I really enjoyed the Probability Lifesaver by Steven J. Miller. It is very thorough, emphasizes the importance of proofs and leads the reader through those proofs, fairly rigorous while leaving out some of the finer details, and helps to think through non-intuitive problems in probability theory. The style is also very accessible and readable.



                                                                        The emphasis on proofs was especially appreciated. I am often tempted to skip proofs since I use probability theory for application and am not a mathematician. A lot of the proofs in this book are described in a style that tries to justify why one would try a particular strategy at each step. The author leads the reader through the proof and tries different approaches, even ones that lead to failure, for the purpose of instruction. This was extremely useful.



                                                                        I enjoyed this book because it gave me the confidence and appreciation of math that has made me more comfortable in reading more advanced and rigorous texts.






                                                                        share|cite|improve this answer
























                                                                          up vote
                                                                          1
                                                                          down vote










                                                                          up vote
                                                                          1
                                                                          down vote









                                                                          This may not be what you are looking for but I really enjoyed the Probability Lifesaver by Steven J. Miller. It is very thorough, emphasizes the importance of proofs and leads the reader through those proofs, fairly rigorous while leaving out some of the finer details, and helps to think through non-intuitive problems in probability theory. The style is also very accessible and readable.



                                                                          The emphasis on proofs was especially appreciated. I am often tempted to skip proofs since I use probability theory for application and am not a mathematician. A lot of the proofs in this book are described in a style that tries to justify why one would try a particular strategy at each step. The author leads the reader through the proof and tries different approaches, even ones that lead to failure, for the purpose of instruction. This was extremely useful.



                                                                          I enjoyed this book because it gave me the confidence and appreciation of math that has made me more comfortable in reading more advanced and rigorous texts.






                                                                          share|cite|improve this answer














                                                                          This may not be what you are looking for but I really enjoyed the Probability Lifesaver by Steven J. Miller. It is very thorough, emphasizes the importance of proofs and leads the reader through those proofs, fairly rigorous while leaving out some of the finer details, and helps to think through non-intuitive problems in probability theory. The style is also very accessible and readable.



                                                                          The emphasis on proofs was especially appreciated. I am often tempted to skip proofs since I use probability theory for application and am not a mathematician. A lot of the proofs in this book are described in a style that tries to justify why one would try a particular strategy at each step. The author leads the reader through the proof and tries different approaches, even ones that lead to failure, for the purpose of instruction. This was extremely useful.



                                                                          I enjoyed this book because it gave me the confidence and appreciation of math that has made me more comfortable in reading more advanced and rigorous texts.







                                                                          share|cite|improve this answer














                                                                          share|cite|improve this answer



                                                                          share|cite|improve this answer








                                                                          edited Sep 3 at 19:43


























                                                                          community wiki





                                                                          2 revs
                                                                          syntonicC





















                                                                              up vote
                                                                              0
                                                                              down vote













                                                                              Probably at more elementary level than you intend, but otherwise (I think) remarkably on target,




                                                                              • Burn, R. P., Numbers and functions: steps into analysis., Cambridge: Cambridge Univ. Press. xix, 328 p. (1993). ZBL0872.00009.

                                                                              (I wish I remembered where I recently saw it recommended. This related question mentions it along with others by the same author.)






                                                                              share|cite|improve this answer


























                                                                                up vote
                                                                                0
                                                                                down vote













                                                                                Probably at more elementary level than you intend, but otherwise (I think) remarkably on target,




                                                                                • Burn, R. P., Numbers and functions: steps into analysis., Cambridge: Cambridge Univ. Press. xix, 328 p. (1993). ZBL0872.00009.

                                                                                (I wish I remembered where I recently saw it recommended. This related question mentions it along with others by the same author.)






                                                                                share|cite|improve this answer
























                                                                                  up vote
                                                                                  0
                                                                                  down vote










                                                                                  up vote
                                                                                  0
                                                                                  down vote









                                                                                  Probably at more elementary level than you intend, but otherwise (I think) remarkably on target,




                                                                                  • Burn, R. P., Numbers and functions: steps into analysis., Cambridge: Cambridge Univ. Press. xix, 328 p. (1993). ZBL0872.00009.

                                                                                  (I wish I remembered where I recently saw it recommended. This related question mentions it along with others by the same author.)






                                                                                  share|cite|improve this answer














                                                                                  Probably at more elementary level than you intend, but otherwise (I think) remarkably on target,




                                                                                  • Burn, R. P., Numbers and functions: steps into analysis., Cambridge: Cambridge Univ. Press. xix, 328 p. (1993). ZBL0872.00009.

                                                                                  (I wish I remembered where I recently saw it recommended. This related question mentions it along with others by the same author.)







                                                                                  share|cite|improve this answer














                                                                                  share|cite|improve this answer



                                                                                  share|cite|improve this answer








                                                                                  answered Sep 2 at 11:15


























                                                                                  community wiki





                                                                                  Francois Ziegler





















                                                                                      up vote
                                                                                      0
                                                                                      down vote













                                                                                      Numbers and Functions by Victor H. Moll and The Concrete Tetrahedron by Manuel Kauers and Peter Paule on an elementary level and Advanced Determinant Calculus (Séminaire Lotharingien Combin. 42 (1999) by Christian Krattenthaler on an advanced level.






                                                                                      share|cite|improve this answer


























                                                                                        up vote
                                                                                        0
                                                                                        down vote













                                                                                        Numbers and Functions by Victor H. Moll and The Concrete Tetrahedron by Manuel Kauers and Peter Paule on an elementary level and Advanced Determinant Calculus (Séminaire Lotharingien Combin. 42 (1999) by Christian Krattenthaler on an advanced level.






                                                                                        share|cite|improve this answer
























                                                                                          up vote
                                                                                          0
                                                                                          down vote










                                                                                          up vote
                                                                                          0
                                                                                          down vote









                                                                                          Numbers and Functions by Victor H. Moll and The Concrete Tetrahedron by Manuel Kauers and Peter Paule on an elementary level and Advanced Determinant Calculus (Séminaire Lotharingien Combin. 42 (1999) by Christian Krattenthaler on an advanced level.






                                                                                          share|cite|improve this answer














                                                                                          Numbers and Functions by Victor H. Moll and The Concrete Tetrahedron by Manuel Kauers and Peter Paule on an elementary level and Advanced Determinant Calculus (Séminaire Lotharingien Combin. 42 (1999) by Christian Krattenthaler on an advanced level.







                                                                                          share|cite|improve this answer














                                                                                          share|cite|improve this answer



                                                                                          share|cite|improve this answer








                                                                                          answered Sep 2 at 16:41


























                                                                                          community wiki





                                                                                          Johann Cigler





















                                                                                              up vote
                                                                                              -1
                                                                                              down vote













                                                                                              If German is an option as well, a rather new book on the market, supplied with more than 1000 youtube videos:



                                                                                              Konkrete Mathematik nicht nur für Informatiker






                                                                                              share|cite|improve this answer


























                                                                                                up vote
                                                                                                -1
                                                                                                down vote













                                                                                                If German is an option as well, a rather new book on the market, supplied with more than 1000 youtube videos:



                                                                                                Konkrete Mathematik nicht nur für Informatiker






                                                                                                share|cite|improve this answer
























                                                                                                  up vote
                                                                                                  -1
                                                                                                  down vote










                                                                                                  up vote
                                                                                                  -1
                                                                                                  down vote









                                                                                                  If German is an option as well, a rather new book on the market, supplied with more than 1000 youtube videos:



                                                                                                  Konkrete Mathematik nicht nur für Informatiker






                                                                                                  share|cite|improve this answer














                                                                                                  If German is an option as well, a rather new book on the market, supplied with more than 1000 youtube videos:



                                                                                                  Konkrete Mathematik nicht nur für Informatiker







                                                                                                  share|cite|improve this answer














                                                                                                  share|cite|improve this answer



                                                                                                  share|cite|improve this answer








                                                                                                  answered Sep 6 at 20:16


























                                                                                                  community wiki





                                                                                                  p6majo




























                                                                                                       

                                                                                                      draft saved


                                                                                                      draft discarded















































                                                                                                       


                                                                                                      draft saved


                                                                                                      draft discarded














                                                                                                      StackExchange.ready(
                                                                                                      function ()
                                                                                                      StackExchange.openid.initPostLogin('.new-post-login', 'https%3a%2f%2fmathoverflow.net%2fquestions%2f309650%2fwhat-are-examples-of-books-which-teach-the-practice-of-mathematics%23new-answer', 'question_page');

                                                                                                      );

                                                                                                      Post as a guest













































































                                                                                                      這個網誌中的熱門文章

                                                                                                      Is there any way to eliminate the singular point to solve this integral by hand or by approximations?

                                                                                                      Why am i infinitely getting the same tweet with the Twitter Search API?

                                                                                                      Solve: $(3xy-2ay^2)dx+(x^2-2axy)dy=0$